Dernier exercice (Cenedra)!

Publié le 31 mai 2010 il y a 13A par Anonyme - Fin › 5 juin 2010 dans 13A
5

Sujet du devoir

Afin de déterminer le prix de deux objets,un menuisier s'interesse à la quantité de bois qu'il à utilisée pour les fabriquer.
L'objet 1 est constituée d'un parallélépipéde rectangle IJKLMNOPQ surmonté d'un prisme droit RSTUVWXY,dont la base est un trapéze.
L'objet 2 est constitué de deux pyramide identiques ayant la même base
On sait que:
FG=4,5dm
La hauteur (EH)issue de E du triangle EFG mesure 4dm

1Montre que :
a.Le volume en dm3 de l'objet 1 est V1=3x+4,8
b.Le volume en dm3 de l'objet 2 est V2=6x

2.a.Le volume V1 est -il proportionnel à x?

b.Le volume V2 est-il proportionnel à x?

3.Sur la feuille millimétré représente graphiquement les fonctions f=3x+4,8 g:6x

4.Détermine la valeur de x pour laquelle V1=V2

Les valeurs de x pour laquelles V1
la valeur de x pour laquelle V1=15;8 dm

5.Résous par le calcul les questions a b et c du 4 précédents!

Où j'en suis dans mon devoir

Désolée pour le retard merciiiiiiiiiiiiiiiiiiiiiiiiiiiiiiiiiiiiiiiiiiiiiiiiiiiiiiiiiiiiiiiii encore pour ton aide



209 commentaires pour ce devoir


5
Anonyme
Posté le 31 mai 2010
Rebonjour,

désolée moi aussi pour le retard....
je vais aller diner et je pense qu'après il sera un peu tard pour travailler des maths!

Essaye de faire ce que tu peux:
-formule
-calcul
-raisonnement
pas forcément complet, juste des pistes

on se donne rendez-vous demain matin vers 9h-10h?
Anonyme
Posté le 31 mai 2010
oui on se donne rendez-vous vers 9h-10h parceque là je suis à bout fatiguée épuisée bref voilà tu es sur que demain tu seras là moi oui comme ça on se pert pas de vue?
Anonyme
Posté le 31 mai 2010
pas de soucis pour demain!
Anonyme
Posté le 31 mai 2010
Ben bonne soirée et merci bien encore bonne appétit et bonne nuit!
Anonyme
Posté le 31 mai 2010
Bonjour Cenedra..

C'est la suite d'un probleme ou l'enonce est etrange?...parce que avec seulement FG = 4,5 .....

Bref j'ai prefere ne pas intervenir. Mais si tu veux et si Kisa est d'accord je peux aussi l'aider si tu est absent...

Bonne soiree.
Anonyme
Posté le 1 juin 2010
Matamore, c'est le dernier exercice d'une série.
Je ne vois aucun inconvénient à ce que tu aides à ma place au contraire!

Kisa, je pense que tu as les figures avec l'énoncé, peux-tu donner les dimensions connues de l'objet 1?
Anonyme
Posté le 1 juin 2010
Bonjour,oui j'ai un schéma!Pour l'objet 1 UT=2,8dm,R'R=1,5 dm
RS=1,2dm pour l'objet 2 les longueurs sont dans l'énonçé!
Anonyme
Posté le 1 juin 2010
Normalement pour le volume en dm3:ce que je sais c'est que l'aire d'un trapèze=(grande base+petite base)*hauteur/2!
J'ai oubliée de préciser que pour l'objet 1 le parallélépipéde rectangle IJKLMNOPQ MN=4dm,PN=5dm je suppose que MQ=5dm aussi KP=2,4dm.........
Anonyme
Posté le 1 juin 2010
Bonjour matamore38,
Merci pour ton aide ça ne me dérange pas que tu m'aide en même temps que Cenedra c'est gentil !
Anonyme
Posté le 1 juin 2010
Kisa, j'ai encore besoin de ton aide :$ en histoire si possible ?
Et je vois, que Cenedra t'aide bien! Puis, moi et les maths, ça fait 2 -_-"
Anonyme
Posté le 1 juin 2010
OK!
Anonyme
Posté le 1 juin 2010
Est-ce-qu'il faut que je trouve l'air du trapèze?
Anonyme
Posté le 1 juin 2010
(4+1,2)*4/2?
Anonyme
Posté le 1 juin 2010
Pour le prisme tu donnes 9 lettres...je suppose qu'il n'y a pas de O. On a donc une base rectangulaire MNPQ; parallele a une base IJKL avec NP = 5; MN = 4; et KP = LQ = IM = JN = 2,4

Pour le prisme avec base trapeze precise quels sont les segments de la grande base; petite base ; et hauteur.....Tu donnes RR' = 1,5..ou est R'?

Pour l'objet 2 precises la forme de la base avec les cotes que tu connais et les points qui sont les sommets des pyramides...

Que represente x dans ton enonce?


Il faut effectivement calculer l'aire du trapeze; puis multiplier cette aire par la hauteur pour trouver le volume du trapeze....
Anonyme
Posté le 1 juin 2010
Bonjour,

la formule de l'aire du trapèze est bonne
tu n'as pas donné le nom des segments formant la grande base, la petite et la hauteur, je ne peux pas vérifier ton calcul
Anonyme
Posté le 1 juin 2010
euh attends 4=MN c'est la grande base
Anonyme
Posté le 1 juin 2010
RS=1,2dm c'est la petite base
Anonyme
Posté le 1 juin 2010
hauteur XT=4dm
Anonyme
Posté le 1 juin 2010
et divisée par 2
Anonyme
Posté le 1 juin 2010
non il n'y a pas de O!
Anonyme
Posté le 1 juin 2010
MN? tu veux dire TU=4dm?

auquel cas:
V(prisme)= (4+1.2)*4/2 est juste
Anonyme
Posté le 1 juin 2010
où est x, du coup?
Anonyme
Posté le 1 juin 2010
je ne sais pas trop pour R' c'est sur le schéma !Tu as la longueur RS=1,2dm et UR et R'est l'angle droit
Anonyme
Posté le 1 juin 2010
x est la hauteur pour le prisme sur mon schéma XT puis il y a une grande fléche sur ce segment et il est marquée xdm
Anonyme
Posté le 1 juin 2010
x représente la hauteur du prisme!
Anonyme
Posté le 1 juin 2010
non TU=2,8dm!
Anonyme
Posté le 1 juin 2010
je me suis trompée pour le nom du parallélépipéde rectangle IJKLMNPQ!
Anonyme
Posté le 1 juin 2010
donc formule pas bonne:
V(prisme)= (UT+RS)/XT/2
avec UT: grande base, 2.8dm
RS: petite base, 1.2dm
XT: hauteur, xdm

V(prisme)= (2.8+1.2)*x/2
Anonyme
Posté le 1 juin 2010
pour l'objet 2 le sommet est en S EFG est un triangle est H est la hauteur qui mesure 4dm!
Anonyme
Posté le 1 juin 2010
(UT+RS)/XT/2 oui je pense que c'est ça!
Anonyme
Posté le 1 juin 2010
erreur de frappe: V(prisme)= (TU+RS)*XT/2
Anonyme
Posté le 1 juin 2010
ah oui,j'avais pas vu maintenant il faut remplaçer par les longueurs?(2,8+1,2)*4/2?
Anonyme
Posté le 1 juin 2010
pourquoi remplaces-tu XT par 4? XT=x, donc tu mets x

V(prisme)= (2.8+1.2)*x/2
Anonyme
Posté le 1 juin 2010
Ah bon?Ah ben oui on ne connait pas x je me suis embrouillée avec l'objet 2!
Anonyme
Posté le 1 juin 2010
STOP!!!!!

on mélange tout!

on n'est pas encore au volume mais à l'aire de base!
Aire trapèze = (grande + petite)*hauteur/2
Volume prisme = aire trapèze * hauteur(du solide)

aire = (TU+RS)*RR'/2
RR' est bien la hauteur du trapèze , RR'= 1.5dm

aire= (2.8+1.2)*1.5/2
aire= .....

volume prisme= aire * XT
avec XT= x

désolée, pas encore bien réveillée...
Anonyme
Posté le 1 juin 2010
non c'est moi qui n'est pas encore bien réveillée!
Anonyme
Posté le 1 juin 2010
aire=(2.8+1.2)*1.5/2
aire=4*1.5/2
aire=6/2
aire=3
Anonyme
Posté le 1 juin 2010
donc, tu trouves:
aire = ....
volume prisme = ....

objet 1 formé du prisme + du parallélépipède
il faut calculer le volume de IJKLMNPQR
Vpara = longueur*largeur*hauteur
Anonyme
Posté le 1 juin 2010
on ne connait pas la longueur XT?
Anonyme
Posté le 1 juin 2010
aire = 3 dm², bravo!

Anonyme
Posté le 1 juin 2010
aire=3x+4.8?
Anonyme
Posté le 1 juin 2010
pour XT, on remplace par x

Vprisme = aire * XT
Vprisme = 3*x = 3x

c'est tout
Anonyme
Posté le 1 juin 2010
Ah ok?
Anonyme
Posté le 1 juin 2010
d'où vient 4.8?

le volume du prisme est de 3x dm³

on passe maintenant au calcul du volume du parallélépipède
Anonyme
Posté le 1 juin 2010
c'est la question a quand il dit il faut prouver V1=3x+4,8 on a trouver 3x maintenant il faut trouver 4,8?
Anonyme
Posté le 1 juin 2010
oui c'est ça!

les mesures sont bien en dm aussi pour IJKLMNPQ?
Anonyme
Posté le 1 juin 2010
oui!
Anonyme
Posté le 1 juin 2010
sauf pour KP=2.4cm sur mon schéma
Anonyme
Posté le 1 juin 2010
il faut convertir KP en dm
KP=2.4 cm = 0.24dm
Anonyme
Posté le 1 juin 2010
d'accord maintenant il faut trouver le volume du parallélépipéde?Il faut faire pareille que le prisme?
Anonyme
Posté le 1 juin 2010
beaucoup plus simple!

Volume para = longueur*largeur*hauteur
longueur: NP
largeur: MN
hauteur: PK
Anonyme
Posté le 1 juin 2010
Ahhhhhhh!ok
Anonyme
Posté le 1 juin 2010
5*4*0,24=4,8 on a réussi à prouver!!
Anonyme
Posté le 1 juin 2010
Bravo!

le volume de l'objet 1 vaut 3x+4.8 dm³

dans l'objet 2, où est x?
Anonyme
Posté le 1 juin 2010
x est EG !
Anonyme
Posté le 1 juin 2010
tu peux reprendre la description de l'objet 2, s'il te plait? je ne vois pas bien la figure..
Anonyme
Posté le 1 juin 2010
pardon x est sur EG!
Anonyme
Posté le 1 juin 2010
L'objet 2 est constituée de 2 pyramides identiques ayant la même base.Les deux pyramides ont pour sommet S .Je ne connais pas la longueur des segments SE,SG S'E S'G EFG forment un triangle FG=4,5dm la hauteur (EH) issue de E du triangle EFG mesure 4dm la hauteur c'est H passe par le sommet S !
Anonyme
Posté le 1 juin 2010
les 2 pyramides ont pour sommet S et S' EFG 1er pyramide en S et EFG 2e pyramide en S'!
Anonyme
Posté le 1 juin 2010
la base commune est donc le triangle EFG
aire de base = FG*EH/2

volume pyramide = aire*hauteur/3

normalement, EH est la hauteur du triangle EFG donc H est sur [FG]
pourquoi dis-tu que H passe par S?
est-ce que [SH] est la hauteur de la pyramide?

est-ce que SH=x (et pas EG)?
Anonyme
Posté le 1 juin 2010
ben sur mon schéma le point h est carrément dans le triangle EFG donc je sais pas trop peut-être qu'il sur FG comme tu dis!
Anonyme
Posté le 1 juin 2010
si on dit dans l'énonçé la hauteur (EH) issue de E du triangle EFG je pense comme toi qu'il doit être sur FG!
Anonyme
Posté le 1 juin 2010
mais non vu que sur mon schéma il la fléche qui montre SH=xdm H passe bien par le sommet S.
Anonyme
Posté le 1 juin 2010
il a tout à fait possible que:
- H soit sur [FG]
- [SH] soit la hauteur

donc pas de problème de ce coté là!

on peut calculer:
aire base = FG*EH/2
aire = 4.5*4/2
aire = ...

volume (EFGS) = aire * SH
volume (EFGS) = aire * x

volume objet 2 = 2* volume (EFGS)
Anonyme
Posté le 1 juin 2010
aire=4.5*4/2
aire=18/2
aire=9
Anonyme
Posté le 1 juin 2010
oui!

attention! j'ai oublié:
volume (EFGS) = aire * SH /3
volume (EFGS)= 9 * x /3
Anonyme
Posté le 1 juin 2010
9*4/3
Anonyme
Posté le 1 juin 2010
pourquoi 4?
SH = x

9*x/3 = 3x
Anonyme
Posté le 1 juin 2010
ah oui?
Anonyme
Posté le 1 juin 2010
oui!

et donc le volume de l'objet 2 = 2*3x = 6x

on trouve la réponse de l'énoncé!
Anonyme
Posté le 1 juin 2010
ah oui je comprends!
Anonyme
Posté le 1 juin 2010
ça veut dire quoi maintenant pour la question 2 a.proportionnel à x!
Anonyme
Posté le 1 juin 2010
deux mesures sont proportionnelles si on peut passer de l'une à l'autre en multipliant ou divisant par un même nombre.

ici nos mesures sont:
- le volume
- x

2a. est-ce que je multiplie x par un nombre (tout seul) pour avoir le volume (3x +4.8)?

2b. est-ce que je multiplie x par un nombre (tout seul) pour avoir le volume (6x)?
Anonyme
Posté le 1 juin 2010
non pour les 2 il faut multiplier par des nombres pour trouver le résultat!!
Anonyme
Posté le 1 juin 2010
je te montre un exemple de proportionnalité:
si une baguette coute 0.50€ alors trois baguettes coutent 1.50€.
j'ai fait prix d'une baguette * nombre de baguettes = 0.50 *3
le prix de trois baguettes est proportionnel au prix d'une baguette.

tu comprend?
Anonyme
Posté le 1 juin 2010
x et volume sont proportionnels si je multiplie l'un par un nombre pour obtenir l'autre

V = 18x, je multiplie x par un nombre (18) donc proportionnel

V = 4x + 23, je multiplie x par un nombre (4) MAIS je rajoute 23 donc ce n'est pas proportionnel
Anonyme
Posté le 1 juin 2010
Ah donc 6x est proportionnel et pas 3x+4.8
Anonyme
Posté le 1 juin 2010
exact!
Anonyme
Posté le 1 juin 2010
c'est vrai j'ai réussi ? merci cenedra!
Anonyme
Posté le 1 juin 2010
Pour la feuille milimétré je dois prendre 2cm pour unité sur l'axe de abscisses (verticale?) axe des ordonnées je dois prendre 1cm(Horinzontale?)
Anonyme
Posté le 1 juin 2010
de rien :)

on passe à la question 3: f(x) et g(x) sont des droites
tu choisis 2 ou 3 valeurs de x (0, 1, 5..) pour avoir les valeurs du volume correspondant
cela te donne les points à placer (x en abscisse (horizontal) et volume en ordonnée (vertical))
tu traces la droite passant par les points

ça va?
Anonyme
Posté le 1 juin 2010
pour x (abscisse, horizontal): 2 cm pour unité
pour V (ordonnée, vertical): 1 cm pour unité

ça me semble bon
Anonyme
Posté le 1 juin 2010
oui attends je place mes points !
Anonyme
Posté le 1 juin 2010
j'ai fait pour x :2cm
Pour V :1cm
Anonyme
Posté le 1 juin 2010
je fais bout par bout comme ça je m'embrouille pas 3x je place mon point sur le 3 des ordonnées?
Anonyme
Posté le 1 juin 2010
pour f(x)= 3x + 4.8

je choisis x=0
f(0)= 3*0 +4.8
f(0)= 4.8

le point est donc sur l'axe verticale à 4.8

je choisis x=1
f(1)= 3*1 +4.8
f(1)= 7.8

le point est donc à l'intersection de x=1 et y=7.8

tu comprend la méthode?
Anonyme
Posté le 1 juin 2010
pas vraiment!
Anonyme
Posté le 1 juin 2010
l'expression 3x+4.8 dépend de x
on ne peut pas calculer le résultat si on ne connait pas x

il nous faut donc donner à x une valeur pour calculer
en fixant x, on obtient f(x) (et f(x) est le volume)

quand je dis x=0, je fixe la valeur de x à 0 (c'est moi qui décide)
je calcule ensuite f(x)
j'obtiens les coordonnées du point que je place.

je prend plusieurs valeurs de x différentes pour pouvoir tracer une droite
Anonyme
Posté le 1 juin 2010
Ah d'accord j'ai compris donc là les points sont sur 1 et 7.8?
Anonyme
Posté le 1 juin 2010
pas exactement, tu te places sur l'axe horizontal à 1, puis tu montes jusqu'à 7.8, le point est là.

Anonyme
Posté le 1 juin 2010
Ah ok!
Anonyme
Posté le 1 juin 2010
ça c'est pour f(x) je fais de meme pour g(x)=6x?
Anonyme
Posté le 1 juin 2010
oui, tout pareil
Anonyme
Posté le 1 juin 2010
donc si je prend 2 pour 6x=2*6=12?
Anonyme
Posté le 1 juin 2010
oui
tu te place sur l'axe horizontal à 2 puis tu montes jusqu'à 12
Anonyme
Posté le 1 juin 2010
Ah j'ai réussi merci encore Cenedra!
Anonyme
Posté le 1 juin 2010
Et c'est tout ensuite je trace la droite?
Anonyme
Posté le 1 juin 2010
ça forme une petite courbe c'est normal?
Anonyme
Posté le 1 juin 2010
normalement c'est une droite pour f(x) et pour g(x)
Anonyme
Posté le 1 juin 2010
donc si je prends 2 c'est pas bon il faut peut être que je prends 1 ou 0?
Anonyme
Posté le 1 juin 2010
pour g(x)=6x????????
Anonyme
Posté le 1 juin 2010
Bon je vois que ca a bien avancé....mais àl'avenir soit plus precise en donna
Anonyme
Posté le 1 juin 2010
Bon je vois que ca a bien avancé....mais àl'avenir soit plus precise en donna
Anonyme
Posté le 1 juin 2010
pour g(x):
x=0 donc g(x)=0 (c'est l'origine du repère)

x=1 donc g(x)=6 (place en 1 tu montes à 6)

x=2 donc g(x)=12 (place en 1 tu montes à 12)

avec ces trois points, tu traces la droite passant par ces trois points
Anonyme
Posté le 1 juin 2010
je peux prendre plusieurs nombre alors?
Anonyme
Posté le 1 juin 2010
erreur x=2 g(x)=12 (place en 2 tu montes à 12)
Anonyme
Posté le 1 juin 2010
il faut en prendre plusieurs pour éviter les erreurs
maximum 4 ou 5 valeurs de x
Anonyme
Posté le 1 juin 2010
mais pour x=3x+4.8=7.8 c'est une droite à part?
Anonyme
Posté le 1 juin 2010
attention à ton écriture!
tu as d'une part f(x)=3x+4.8 (fonction qui donne le volume de l'objet 1)

tu as d'autre part g(x)=6x (fonction qui donne le volume de l'objet 2)

sur le graphique tu as donc deux droites.

quand on s'occupe de tracer f(x), je prend par exemple x=1, j'obtiens f(1)= 3+4.8=7.8
cela me donne un point
je dois obligatoirement prendre un autre point pour tracer une droite (deux points c'est le minimum)
pour cet autre point, je prend x=5, j'obtiens f(5)=3*5+4.8= 19.8

là, je peux tracer la droite qui passe par ces deux points.

quand on s'occupe de tracer g(x), je prend par exemple x=0, j'obtiens g(0)=0
le point obtenu est l'origine du repère
je dois prendre un autre point, je prend x=2, j'obtiens g(2)=12

là je peux tracer la doite qui passe par ces deux points.


est-ce que dit comme ça, c'est plus clair?
Anonyme
Posté le 1 juin 2010
oui j'ai compris!
Anonyme
Posté le 1 juin 2010
pour g(x)=6x je suis pas obligée de prendre 0 j'ai pris 1 et 2 c'est bon?de toute façon il passe par l'origine du repére?
Anonyme
Posté le 1 juin 2010
tu n'es pas obligée de prendre 0 (c'est juste facile)
et de toute façon la droite passe par l'origine.

tu as réussi à tracer?
Anonyme
Posté le 1 juin 2010
si je prends comme toi 7.8 comme point et 19.8 comme 2e points il ne passe pas par l'origine du repère!
Anonyme
Posté le 1 juin 2010
pour 1et 2 oui pour 6x oui j'y suis arriver à tracer la droite!
Anonyme
Posté le 1 juin 2010
f(x) ne passe pas par l'origine du repère, c'est normal

f(x) est dite fonction affine
g(x) est dite fonction linéaire (car il y a proportionnalité)
Anonyme
Posté le 1 juin 2010
ah bon donc c'est pas grave si il ne passe pas par l'origine du repére?c'est ça que je voulais savoir en faite merci cenedra!
Anonyme
Posté le 1 juin 2010
et la droite 7.8 et 19.8
Anonyme
Posté le 1 juin 2010
pardon elle coupe la droite qui passe par l'origine du repére c'est normal?
Anonyme
Posté le 1 juin 2010
Oui excuse moi!
Anonyme
Posté le 1 juin 2010
g(x) passe par l'origine , c'est facile

f(x) ne passe pas par l'origine, elle coupe l'axe des ordonnées (vertical) à 4.8

si tu as ça alors c'est bon!
Anonyme
Posté le 1 juin 2010
moi non elle coupe en 1.
Anonyme
Posté le 1 juin 2010
j'ai fait coupée la droite en 4.8?
Anonyme
Posté le 1 juin 2010
mais ce n'est pas une droite?
Anonyme
Posté le 1 juin 2010
va voir ce lien:
http://yfrog.com/0zgraphvrp

j'ai tracer les droites, tu dois donc avoir ça
en bleu f(x)
en rouge g(x)
Anonyme
Posté le 1 juin 2010
d'accord merci tu es vraiment quelqu'un de bien.
Anonyme
Posté le 1 juin 2010
il me dit aucun lien trouvée c'est normale?
Anonyme
Posté le 1 juin 2010
euh, non ça c'est pas normal, je vérifie
Anonyme
Posté le 1 juin 2010
si le lien ne marche toujours pas, essaye avec celui-là:

http://img35.imageshack.us/img35/363/graphvr.png
Anonyme
Posté le 1 juin 2010
Merci c'est gentil j'essaie de comprendre comment est la droite f(x)!
Anonyme
Posté le 1 juin 2010
non toujours pas mais bon c'est pas grave!
Anonyme
Posté le 1 juin 2010
je crois que j'ai trouvée!il fallait mettre sur 4;8 le point en premier puis 7.8 et 19.8 puis tracer la droite?
Anonyme
Posté le 1 juin 2010
euh, oui, les trois points doivent être alignés
Anonyme
Posté le 1 juin 2010
Cenedra j'ai trouvée c'est bon je m'excuse c'est moi qui avait mal plaçée un point c'est bon j'ai pu traçée les deux droites!
Anonyme
Posté le 1 juin 2010
Merci !!!
Anonyme
Posté le 1 juin 2010
mais ça va j'ai compris!
Anonyme
Posté le 1 juin 2010
pfouuu, yes!

question 4:
graphiquement V1=V2 correspond au point d'intersection des deux droites
ça te semble logique?

la réponse attendue est la valeur de x de ce point
Anonyme
Posté le 1 juin 2010
oui ça veut dire que les résultats correspond?
Anonyme
Posté le 1 juin 2010
cela veut dire que le volume de l'objet 1 est égal au volume de l'objet 2
Anonyme
Posté le 1 juin 2010
Ah ok!
Anonyme
Posté le 1 juin 2010
je dois déterminer sur le graphique!V1=V2
Anonyme
Posté le 1 juin 2010
oui

et tu trouves x= ...
Anonyme
Posté le 1 juin 2010
c'est la que je bloque!
Anonyme
Posté le 1 juin 2010
x=il faut pas dire toutes les valeurs?
Anonyme
Posté le 1 juin 2010
il n'y a qu'une seule valeur de x pour V1=V2

je rappelle:
f(x)=3x +4.8 = V1
g(x)= 6x = V2

dire V1=V2 signifie graphiquement que l'on cherche f(x)= g(x)
il s'agit du croisement des deux droites: c'est un point
on veut le x de ce point
Anonyme
Posté le 1 juin 2010
je suis entrain de dire des betise la!
Anonyme
Posté le 1 juin 2010
x=9.5
Anonyme
Posté le 1 juin 2010
non ce n'est pas ça

comment tu sais que tu va dire une bêtise?
Anonyme
Posté le 1 juin 2010
c'est parceque je suis fatiguée que je dis des bêtises!
Ben pourtant ça se croise?
Anonyme
Posté le 1 juin 2010
les deux droites se croisent, c'est certain

attend, 9.5 c'est y!
tu lis sur l'axe vertical 9.5 pour le point

les x sont sur l'axe horizontal, donc x= ...
Anonyme
Posté le 1 juin 2010
x=1.5
Anonyme
Posté le 1 juin 2010
ou plutôt 1.6 ou 1.7
Anonyme
Posté le 1 juin 2010
je suis d'accord
on peut lire sur le graphique de V1=V2 pour x=1.5

la question suivante, je n'ai pas la fin: il faut lire graphiquement que V1....?
Anonyme
Posté le 1 juin 2010
si tu lis x= 1.6 alors on prend 1.6 et pas 1.5
Anonyme
Posté le 1 juin 2010
c'est un petit carré aprés donc 1.6
Anonyme
Posté le 1 juin 2010
b.détermine graphiquement de x pour lesquelles V1
Anonyme
Posté le 1 juin 2010
tu dois donner un ensemble de valeurs de x pour lesquelles V1 c'est à dire f(x)
Anonyme
Posté le 1 juin 2010
je comprends plus trop!
Anonyme
Posté le 1 juin 2010
4a. V1=V2 réponse x= 1.6

4b. V1 pour savoir on regarde les courbes
V1 = f(x)
V2 = g(x)

si V1
Anonyme
Posté le 1 juin 2010
il y 1 qui est + petit
Anonyme
Posté le 1 juin 2010
1 ne fait pas partie de la bonne réponse

regarde tes droites, sur une zone, f(x) est au dessus puis sur une autre zone, g(x) est au dessus
cette zone = intervalle de x (valeur de l'axe horizontal)

tu dois donner l'intervalle de x où f(x) est en dessous de g(x) (ou g(x) au dessus de f(x), c'est la même chose)
Anonyme
Posté le 1 juin 2010
il y a 4,3 c'est bon?
Anonyme
Posté le 1 juin 2010
pour x= 4.3, oui mais pas seulement
on attend un intervalle,
par exemple: [0; 1.6[ est l'intervalle où g(x) est en dessous de f(x)
Anonyme
Posté le 1 juin 2010
(0;4) (0;3) (0;2)
Anonyme
Posté le 1 juin 2010
pour x compris entre 0 et 1.6, g(x) est en dessous de f(x) donc V2
est-ce que pour x compris entre 0 et 4, f(x) est en dessous de g(x)?
Anonyme
Posté le 1 juin 2010
euh non!
Anonyme
Posté le 1 juin 2010
donc ce n'est pas la réponse!

f(x) est en dessous de g(x) pour x compris entre ... et ...?
Anonyme
Posté le 1 juin 2010
2 et 3
Anonyme
Posté le 1 juin 2010
suelement entre 2 et 3?
Anonyme
Posté le 1 juin 2010
aprés je vois pas!
Anonyme
Posté le 1 juin 2010
peut-être 0et1?
Anonyme
Posté le 1 juin 2010
je te donne la réponse:
pour x compris entre 1.6 et l'infini
]1.6 ; + infini[

à partir du point d'intersection à x= 1.6 et tout ce qui a à droite correspond à la zone où V1
Anonyme
Posté le 1 juin 2010
Ah d'accord!ben j'avais vraiment pas vu!
Anonyme
Posté le 1 juin 2010
4c. V1=15.8 dm³
tu regarde l'axe des ordonnées à 15.8
la droite f(x) atteint 15.8 pour quelle valeur de x?
Anonyme
Posté le 1 juin 2010
x=2.6
Anonyme
Posté le 1 juin 2010
je ne trouve pas 2.6 mais 3.6
Anonyme
Posté le 1 juin 2010
c'est moi qui a fait une betise parceque j'ai mis le point sur la droite ou les points sont 4.8,7.8 etc il faut mettre sur l'autre?
Anonyme
Posté le 1 juin 2010
non c'est bien sur cette droite là
est-ce que tu pars bien de 15.8?
Anonyme
Posté le 1 juin 2010
oui!et je trouve 2.6
Anonyme
Posté le 1 juin 2010
on parle bien de la droite qui ne passe pas par l'origine du repère?
Anonyme
Posté le 1 juin 2010
oui!
Anonyme
Posté le 1 juin 2010
et c'est toi qui a raison c'est 3.6
Anonyme
Posté le 1 juin 2010
ouf!

je reprend:
4a. V1=V2 pour x= 1.6
4b. V1 4c. V1=15.8 pour x=3.6

nous avons trouvé ces résultats graphiquement

la question 5: même chose mais algébriquement
c'est-à-dire avec les équations

5a. V1=V2 signifie f(x)=g(x)
donc: 3x + 4.8 = 6x
ce qui donne : ......?
Anonyme
Posté le 1 juin 2010
15.8+3.6?
Anonyme
Posté le 1 juin 2010
pourquoi 15.8 + 3.6?

il faut juste résoudre
3x +4.8 = 6x

(résoudre veut dire donner x=...)
Anonyme
Posté le 1 juin 2010
Ah ok!
Anonyme
Posté le 1 juin 2010
mais comment faire pour résoudre il faut diviser ?
Anonyme
Posté le 1 juin 2010
3x + 4.8 = 6x

je passe les x d'un coté
3x + 4.8 -3x = 6x - 3x
4.8 = (6-3)x
4.8= 3x
4.8/3 = 3x/3
1.6 = x

on retrouve la réponse du 4a!
Anonyme
Posté le 1 juin 2010
Ah oui j'avais oubliée ça!
Anonyme
Posté le 1 juin 2010
il faut faire pour le b pareil?
Anonyme
Posté le 1 juin 2010
oui

il faut résoudre l'inéquation
V1 donc: 3x + 4.8 < 6x
donc: ...
Anonyme
Posté le 1 juin 2010
la aussi il faut changer x de coté ?
Anonyme
Posté le 1 juin 2010
oui
Anonyme
Posté le 1 juin 2010
3x+4.8<6x
3x+4.8+3=10.8-6=4.8 c'est pas bon?
Anonyme
Posté le 1 juin 2010
tu perd les x au passage...

3x + 4.8 < 6x
3x + 4.8 - 3x < 6x - 3x
4.8 < 3x
4.8/3 < 3x/3
1.6 < x

donc pour tous x supérieur à 1.6, V1 < V2

on retrouve encore le résultat du 4b.

5c.
V1 = 15.8
donc: 3x + 4.8 = 15.8
à toi!
Anonyme
Posté le 1 juin 2010
3x+4.8=15.8
3x+4.8-3x=15.8-3x
dis moi si c'est bon le début?
Anonyme
Posté le 1 juin 2010
il n'y a pas d'erreur mais ça ne sert pas à grand chose de bouger 3x
fais la même chose avec 4.8
Anonyme
Posté le 1 juin 2010
donc tu veux dire que je dois au lieu de mettre -3x je mets 4.8?
Anonyme
Posté le 1 juin 2010
au lieu de faire:
3x +4.8 -3x = 15.8 -3x

tu fais:
3x + 4.8 - 4.8 = 15.8 - 4.8
Anonyme
Posté le 1 juin 2010
Ah d'accord!
3x+4.8-4.8=15.8-4.8
3x/15.3 je comprends plus!
Anonyme
Posté le 1 juin 2010
si j'ai trouvée 15.8-4.8=11/3=3.66
Anonyme
Posté le 1 juin 2010
c'est bon non?
Anonyme
Posté le 1 juin 2010
bravo!

plus proprement:
3x + 4.8 = 15.8
3x + 4.8 - 4.8 = 15.8 - 4.8
3x = 11
3x/3 = 11/3
x= 3.66

on retrouve le même résultat que 4c.
Anonyme
Posté le 1 juin 2010
l'exercice est terminée ou pas?(je veux juste m'assurer que tout est bien fait)!
Anonyme
Posté le 1 juin 2010
Bravo à toi, l'exercice est terminé!
Anonyme
Posté le 1 juin 2010
OHHHHHHHHHHHHHH MERCIIIIIIIIIIIIIIIIII à TOIIIIIIIIIIIIIIIIIIIII tu as été vraiment formidable tu as été trés patiente envers moi et j'ai jamais trouvée une personne qui soit vraiment patiente !
Merciiiiiiiiii encore Cenedra merci infiniment j'en ai finie avec les Maths!
Anonyme
Posté le 1 juin 2010
Merciiiiiiiiiiiiiiiiiiiiiiiiiiii!!!!!!!!!!!!!Beaucoupppppppppppppppppppppppppppppppp!!!!!!!!!!!!
Merciiiiiiiiiiiiiiiiiiiiiiiiiiiiiii!!!!!!!!!!!
Anonyme
Posté le 1 juin 2010
merciiii encore!!!!
Anonyme
Posté le 1 juin 2010
de rien!

félicitations à toi d'avoir compris et travaillé toute la journée!
Anonyme
Posté le 1 juin 2010
merciiiiiiiiiiiiiiiiiiiiiiiiiiiiiiiiiiiiiiiiiiiiiiiiii!!!!!!!!!!!!!!!!!!!!!!!!!!!!!!!!!!!!!!!!!!!!!!!!

Ils ont besoin d'aide !

Il faut être inscrit pour aider

Crée un compte gratuit pour aider

Je m'inscrisOU

J'ai déjà un compte

Je me connecte